Try the followings:
Consider a prism with a triangular base. The total area of the three faces containing a particular vertex is . Show that the maximum possible volume of the prism is and find the height of this largest prism.
Consider an grid divided into subgrids of size . Find the number of ways in which you can select cells from this grid such that there is exactly one cell coming from each subgrid, one from ach row and one from each column.
If are not necessarily distinct reals, show that we can choose three of them such that they are lengths of the sides of a triangle.
Show that there cannot exist a non-constant polynomial such that is prime for all positive integers .
(Calculus) Let be a twice differentiable function on the open interval such that . Suppose that also satisfies , and , for all . Show that .
Easy Math Editor
This discussion board is a place to discuss our Daily Challenges and the math and science related to those challenges. Explanations are more than just a solution — they should explain the steps and thinking strategies that you used to obtain the solution. Comments should further the discussion of math and science.
When posting on Brilliant:
*italics*
or_italics_
**bold**
or__bold__
paragraph 1
paragraph 2
[example link](https://brilliant.org)
> This is a quote
\(
...\)
or\[
...\]
to ensure proper formatting.2 \times 3
2^{34}
a_{i-1}
\frac{2}{3}
\sqrt{2}
\sum_{i=1}^3
\sin \theta
\boxed{123}
Comments
Last one:
There must be a value of x∈(0,1) satisfying f′(x0)=1−0f(1)−1
For this x , (f′(x0))2=(1−f(1))2
Now,
f′′(x)≤f(x)⇒f′(x)f′′(x)≥f′(x)f(x)
Hence,
d((f′(x))2)≥d((f(x))2)
Integrate in limits 0 to x0 to get:
(f′(x0))2−(f′(0))2≥(f(x0))2−1
Hence, (f′(0))2≤1+(1−f(1))2−(f(x0))2
Clearly, (f′(0))2≤2 ,or f′(0)≥−2
Log in to reply
Good!
hey paramjeet i cant get it can u explain me in detail
Log in to reply
What needs explanation? And it's Paramjit. :)
Problem 3 is essentially USAMO 2012 Problem 1 in disguise. Can you see why?